Anda di halaman 1dari 39



91

Indian Financial
System

This Chapter ‘‘Indian Financial


System” is taken from our Book:

ISBN : 9789386146977
92 Indian Financial System

Indian Financial
8
Chapter
System
Introduction This system provides detailed information
A well established financial system about the players in the market such as
plays very important role in economic individuals, corporate houses, government
agencies etc.
development of any country. A financial
It also provides a mechanism for controlling
system consists of financial institutions,
risks involved in managing savings and
financial markets, financial instruments and
allocating funds.
financial services. This system provides a
It covers the whole gamut of demand for and
framework by which savings and surplus supply of funds for productive purposes.
funds are mobilized in a productive manner. The financial system promotes economic
A financial system servers as a link between development through mobilising savings
savers and investors and channelising these to investment
It promotes the capital formation by avenues.
bringing together supply of savings and The Indian financial system consists of both
demand for funds. short term and long term finances.

MONEY MARKET short – term money market. Call money


transactions are limited between a day
Call Money Market and a fortnight and are most applied in
• A money market, which involves the case of inter – bank exchanges.
financial transaction (lending and • Call Money, Notice Money and Term
borrowing) for only a small period of Money markets are vital components of
time, is termed as Call Money Market, or the Indian Money Market.
Indian Financial System 93

• Call money involves monetary competitive wherein the bidder needs to


transaction for a day; notice money refers specify the expected return.
to the borrowing and lending of funds for • Individuals, Firms, Trusts, Institutions
2 – 14 days and term money refers to and banks can purchase T-Bills.
financial transactions with a time frame
• The advantages of such bills are as
exceeding a fortnight.
follows:
• For such markets, the interest rates are (a) Zero Risk: T–Bills are issued by the
subjected to the market conditions and Government and thus, the investor
bizscapes. For instance, in India, the has no botheration
public sector banks account for 80% of (b) High Liquidity: Short–term
the demand whereas the foreign and the investments (3 months, 6 months,
private players result for the remaining 9 months)
20%. To cater to these demands, (c) Transparency: Regulated by the
institutions like IDBI and LIC supply Government.
majority of the short term funds to the
(d) High Tradability: The secondary
state banks vis – a - vis other banks. market of T–Bills is highly
• Since banks feature as lenders and organized
borrowers in this process, it is called • The Central Government of India
Inter – Bank Market. issues such bills, for a minimum
• Call money is mostly liquid money amount of `25, 000 and in multiples
and policies are framed with regards of the same.
to the RBI intervention. These short– Commercial Bill Market
term policies are located in established
• The commercial bill market relates to
cities like Chennai, Kolkata, Mumbai
the seller and buyer equation over the
and Delhi.
purchased goods.
Treasury Bill Market • Commercial bills are issued by the
• Short–term securities that mature within seller (drawer) on the buyer (drawee)
an year from their issue date are called for the value of the goods or products
Treasury Bills or T – Bills. These delivered by him.
policies are effectively deployed by the • Commercial bills are considered as
US Government to raise money from the marketable investments. The process
public. involves a seller (in need of funds)
• Initially, T – Bills are purchased at prices sending a bill to the buyer, who in
lower than their par (Face) value and after turns accepts the same and promises
maturity, the government pays the bearer, on – time payment. The bank can
the full par value. Mathematically, the also be approached to accept the bill.
interest is actually the difference between The bank levies a commission for the
the purchase price of the security and the acceptance of the bill and vows to pay
amount received post maturity. if the buyer defaults. Following this
protocol, the seller can sell his goods
• A competitive bidding or auction process
in the market.
is employed to issue T – Bills; either
non – competitive wherein the return is • Such bills are instrumental in providing
specified post the bidding process and short-term financial impetus to
94 Indian Financial System

businesses. However, these bills failed Commercial Paper Market


to be effective as cash credit scheme
• Commercial paper is an unstructured
is the prime form of bank lending and
or rather unsecured bond, issued by a
big corporate firms do not abide by the
corporation, based on receivables and
principle of timely payments.
inventories. Maturities on such bonds
• The difference between the commercial are no longer than nine months.
and the T-bill is that the latter is issued
by the Government whereas the former • These alternatives are actually targeted
is imposed by the seller. by organizations, aiming at borrowing
short–term money from banks. The
Collateral Loan Market conventional process, being very
• Providing and availing loans is a tedious and process–oriented, has led
primary factor of the financial market. to the fame of commercial paper.
In those cases where the principal • This method is otherwise very safe,
amount of the loan is in massive
as it easily indicates the financial
proportion, the lender (banks mostly)
condition of a company within a few
imposes a collateral on the borrower.
months. There has not been many cases
• Collateral represents the asset which
of defaulters in the last four decades,
can be pledged, as a security to the
because commercial papers are issued
creditor by the borrower. The collateral
amount depends according to the value to companies with high credit ratings
of the loan. and good reputation only.
• The collateral policies are implemented • The commercial paper market provides
in case of real estate purchase or car a means for corporations to borrow
purchase. Here, the property itself or money to cover short-term debt
the vehicle will act as collateral, until obligations, such as payroll.
the loan is paid in full.
• Complete information related to Money Market Instruments
such loans is furnished in a contract, • Chakravarthy committee and Narasimhan
which is signed by the lender and the committee recommended certain money
borrower. market instruments to reform Indian
• For loans on vehicles, the vehicle itself money market. Some of these are:
is kept as security. In the condition of
• 182 days treasury bills which are sold
a default in the borrower’s repayment,
the vehicle would be legally seized by through fortnightly auctions. They
the financial institution, which it is carry attractive rates of interest and
hypothecated to. practically no risk and are therefore
popular with commercial banks.
• Jewelry and other securities can also
be used as collateral in giving loans. • 364 days treasury bills were also
In such cases, the ownership of these introduced in 1992.
securities remains with the borrower • Dated government securities with
but in case of a failure to repay, the maturities up to 10 years have also
ownership rights are transferred to the been introduced primarily to develop
bank. a secondary market.
• The collateral process provides a level • Money market mutual funds have been
of confidence and assurance at the permitted to be floated by commercial
time of giving the loan. banks.
Indian Financial System 95

• Repurchase options (repos) and securities are called gilt or gilt edged
reverse repos have been introduced in securities.
order to even out sharp fluctuations • Should there arise a situation where
in the money market. Repos provide in the Government creates a security
an opportunity for RBI to repurchase for raising a public loan, an intimation
government securities from regarding the same is notified in the
commercial banks. Reverse repos are Official Gazette under the Government
government securities sold through
Securities Act, 2016.
auction at fixed cut-off rate of interest.
• Gilt–edged securities are a high –
• Liquidity Adjustment Facility (LAF)
grade investment with very low risk.
refers to RBI’s policy of using Repos
and Reverse Repos to adjust liquidity • High–grade bonds can also be issued
on a day-to-day basis. by private firms too, which flaunts a
long record of consistent earnings and
CAPITAL MARKET possess ability to pay its obligations on
Capital market deals with long-term time and not accrue any bad debts in
finance (more than 365 days) funds. It business transactions.
includes all facilities and institutional • The term, ‘gilt–edge’ initially
arrangements available for borrowing originated from Britain; then referred
and lending of term funds (including to the debt securities issued by the
medium-term). Bank of England, on behalf of His/ Her
The difference between money market Majesty’s Treasure.
and capital market is not so much in • Depending upon expiry date, government
the institutions involved as in their term securities are classified into the following:
of borrowing or lending. Long-term
funds are raised either by borrowing Short – term gilt: Long – term gilt:
from certain institutions or by issuing Maturity: Maturity:
securities. >1 year 5 /10 /15 yrs.
The main players in Indian capital • Additionally, these gilt–edged securities
markets are: provide safety due to the zero income
• Banks, indigenous and commercial. default, 100% liquidity and bulk
• Insurance companies investment opportunity owing to the
high rate of return.
• Development Finance Institutions
(DFI), (b) Corporate
• Non-Banking Finance Companies, • Corporate security  identifies and
(NBFCs) effectively mitigates, at an early
• Non-Banking Financial Institutions. stage, any developments that may
The capital market may be divided into threaten the resilience and survival of
a corporation. It is a corporate function
(i) the securities market; and (ii) the that oversees and manages the close
financial institutions. coordination of all functions within
Securities the company that are concerned with
security, continuity and safety.
(a) Gilt–edged • Corporate securities or company
• There are securities issued by the securities are known to be the
government to borrow money from documentary media for mobilising
the market. These government issued funds by the joint stock companies.
96 Indian Financial System

• The need for corporate securities arises • Creditorship securities are also called
in the following: debentures and accounts for the debt
(a) Successful establishment of of a company. Debenture holders
business functions are regarded as the creditors of the
company and debentures account for
(b) Sponsoring of fund–intensive
the borrowed capital. A debenture may
expansion plans
be defined as an instrument executed
• There are two types of corporate by company under its common seal,
securities: acknowledging indebtedness to an
Ownership Creditorship individual or a group, to secure the
Securities Securities sum advanced. Debentures are usually
bonds issued by the company in series
• Ownership securities consist of of a fixed denomination e.g., `100,
preference shares and equity shares. `200, `500, `1,000 of face value and
Preference shares are those shares are offered to the public, by means of
which carry priority rights related a prospectus.
to dividend payment at a fixed rate
and repayment of capital, in the Financial Institutions
event of a company being wound (a) SEBI
up. The advantages of preference
• Established in 1988 and provided
shares include mobilizing funds from statutory powers in 1992, the Securities
investors who prefer stable earnings and Exchange Board of India (SEBI)
with assurance, flexibility in capital is the regulator for the Indian security
structure as desired, complete control market.
of business transactions within an • The Indian Parliament passed the SEBI
organization and increase in the profits Act on 12 April, 1992.
of the shareholders. Disadvantages
• SEBI is headquartered in Mumbai,
include not allowing investors to carry
Maharashtra.
voting rights, shares being expensive,
• Before the Government of India
income tax problems and redemption
enforced the existence of SEBI,
issues at the time of depression.
Controller of Capital Issues was the
• Equity shares are ordinary shares, regulatory authority.
devoid of special attributes with • The main motto behind constructing
respect to dividend or return of SEBI was to regulate and control the
capital, as in preference shares. Equity function of capital markets in India
shareholders are the residual claimants under the intervention of the Indian
against the assets and income of the Government.
corporation.”The financial risk is • The SEBI is managed by: (a) The
more with equity share capital, also Chairman – nominated by the Union
called ‘risk capital.’ Some of the Government of India; (b) two officers
advantages of these shares are long from the Union Finance Ministry; (c)
shelf – life of funds, shareholders’ one member from the RBI and five
right to participation in the affairs of members nominated by the Union
the company, increase in shareholders’ Government, among which three
assets and ownership rights of the should be whole – time members.
shareholders.
Indian Financial System 97

• The present Chairman of SEBI is Mr. entrepreneurs and small businesses,


Upendra Kumar Sinha, who replaced especially in the semi – urban areas
C. Bhave in 2011. which lack access to banks and related
• The Preamble defines SEBI’s services; community development
immediate responsibility to protect financial institution which provides
the interests of the investors, promote credit and financial services to the
goodwill, usher development and deprived markets and populations
regulate proceedings pertaining to the and revolving loan funds, which
assists micro, small, medium and
securities market.
rural projects by providing loans to
• SEBI caters to the needs of the security individuals who does not otherwise
issuers, the investors and the market qualify for conventional financial
intermediaries. benefits.
• SEBI also has additional • Some of the important characteristics
responsibilities to draft SOPs and of these institutions are providing
regulations (legislative), conducts credit in the form of higher risk loans
investigation and verification for and equity positions.
proper enforcement (executive) and • DFIs are commonly seen in the
passes rulings and orders (judicial) developed countries, supported by the
• In order to streamline, regulate and states.
monitor its duties, the SEBI has been • For markets with severe restriction
bestowed with the following powers: and lack of financial access, DFIs are
(a) To approve by–laws of stock very useful for providing finance for
exchanges; inclusive growth and development.
(b) To instruct stock–exchanges to (c) IFCI
modify their by–laws;
• In 1947, shortly after the Independence,
(c) To inspect the accounts and ledgers it was observed the India’s capital
and call for periodical returns from market was relatively underdeveloped
major stock exchanges; due to lack of policies, benchmarks
(d) To inspect the accounts and ledgers and service providers. However, the
of financial intermediaries; demand was relatively high. To add
to the woes, there were no merchant
(e) To register brokers after validating
bankers or underwriting firms and
their background verification.
neither were proper commercial banks
• Some of the important SEBI to provide long – term investment
Committees are the Primary Market options or portfolios.
Advisory Committee (PMAC),
• Against such a backdrop, the Industrial
Secondary Market Advisory
Finance Corporation of India (IFCI)
Committee (SMAC), Mutual Fund was constituted on July 1, 1948. IFCI,
Advisory Committee and Advisory at its inception was meant to provide
Committee for the SEBI Investor access to cost – effective funds through
Protection and Education Fund. the Central Bank’s Statutory Liquidity
(b) DFI Ratio (SLR).
• A Development Finance Institution (DFI) • IFCI, thus became an India Government
is a subsidiary financial establishment owned development bank to provide
which includes microfinance institutions long – term financial leverage to the
– agencies which sponsor budding industrial sector.
98 Indian Financial System

• IFCI’s contribution to the modernization India and is globally present in 19


of Indian Industry, export promotion, countries.
import substitution, entrepreneurship • Along with giants like SBI, PNB and
development, pollution control, energy BoB, ICICI is noted among the big
conservation and generation of both four banks of India.
direct and indirect employment is
• ICICI Bank was established by
noteworthy.
the  Industrial Credit and Investment
• IFCI was reinstated as an organization Corporation of India (ICICI), an
in 1993 (under the Companies Act, Indian financial institution, as a
1956), to impart higher degree of wholly owned subsidiary in 1994. The
operational benefits, and access the parent company was formed in 1955
capital markets directly. as a joint-venture of the  World Bank,
• With effect from 1999, IFCI changed India’s public-sector banks and public-
to IFCI Limited. sector insurance companies to provide
project financing to Indian industry
• The modus operandi of IFCI Limited
was to facilitate provision for medium • ICICI was the first Indian bank to
and long term financial support to be enlisted under the NYSE in 1999,
large scale industries, especially when being a non – Japanese institution.
banks do not have the authority for an • Over the last two decades, ICICI has
undertaking or issuing shares. witnessed several important mergers
• Some of the vital responsibilities with other banks and private partners
to consolidate its business foundation.
include providing loans and advances
to major industrial projects, facilitating • As a stalwart, it influences regulators
loan sanction in domestic and such as the National Stock Exchange,
international currencies, underwriting the Credit Rating Information System
the issue of stocks, bonds and shares. of India Limited, National Commodities
and Derivatives Exchange Ltd. and
(d) ICICI NABARD.
• ICICI, an acronym for Industrial • Some of the vital portfolios include
Credit and Investment Corporation ‘Money2India’ – an online money
of India, is a multinational banking transfer and tracking facility provided
and financial services company , based to non resident Indians by the bank,
out of Mumbai and registered office in ‘Extra Home Loans’ – mortgage –
Vadodara. guaranteed supported loans for retail
• Over the years since inception, customers aiming at purchasing
ICICI Bank has accumulated several their homes in the economical
accolades. In 2014, it was declared housing segment, ‘Smart Value’ – the
as the second largest bank in terms automated system of 24X7 lockers,
of assets and third in terms of market including weekends and wee hours;
‘Saral Loans’ – to provide loans at
capitalization.
nominal rate of interests to the rural
• Out of the vast portfolio of products folks, including women; ‘Video
and services, few worth mentioning Banking for NRIs’; ‘Contactless Debit
are investment banking, life insurance, and Credit Cards’ and ‘iWish’ – the
venture capital and asset management. flexible recurring deposit scheme
• The ICICI empire has a network of to allow customers deposit feasible
4450 branches and 13995 ATMs in amounts of their choice each month.
Indian Financial System 99

• ICICI is a brand known for its long – term financing requirements,


CSR initiatives like the ‘Go Green under the RBI guidelines.
Initiative’, ‘Jiyo Khulke’ contest and • In 1976, the statutory ownership
‘Read to Lead.’ of IDBI was transferred to the
• A noble lady, a visionary, ICICI’s CEO, Government of India, which was
Chanda Kochhar is one of India’s most initially a wholly–owned subsidiary of
powerful corporate tycoon, of recent the RBI, since the inception in 1964,
times and has been influential in under an Act of the Parliament.
creating the success story. Some of her
• IDBI can be accounted for the various
milestones are appropriate case studies
reforms during the 1964–1991 period,
for inspiring the youth.
and has assisted in setting institutions
• Under Kochhar’s leadership, ICICI has like the Securities and Exchange
won the title of the ‘Best Retail Bank Board of India (SEBI), National
in India’ thrice consecutively. Stock Exchange of India (NSE), the
• However, amidst such appreciation, EXIM Bank and the Small Industries
ICICI has been criticized for several Development Bank of India (SIDBI).
money laundering scams and inhuman
• With the Industrial Development Bank
debt recovery methods using goons.
(Transfer of Undertaking and Repeal)
(e) IDBI Act, 2003, IDBI was bestowed with
• The Industrial Development Bank of the status of a limited company viz.,
India (IDBT), as it was formerly known, IDBI Ltd. Shortly thereafter, IDBI was
is a government–owned financial service incorporated as a ‘scheduled bank’
company, headquartered in Mumbai. under the RBI Act, 1934.
• The main motto behind its establishment • IDBI, just like ICICI has been
was to supply credit and financial crowned with many jewels; one worth
stability to the Indian industrial sector. mentioning is Dun & Bradstreet rating
• With 1853 branches and 3350 ATMs of the ‘Best Public Sector Bank’ in
and is a significant player under the 2011, a period where customers were
aegis of commercial banks owned by dissatisfied with the offerings of the
the Indian Government. government banks.

• The Bank has an aggregate balance • Mr. Kishor Kharat is captaining the
ship, being the CEO and MD.
statement of INR 3.74 trillion, at the
closure of the last Financial Year. • Some of the important portfolios
include consumer banking, corporate
• IDBI is classified as a development
banking, investment banking,
bank. Turning the pages of history,
mortgage loans, wealth management
development banking emerged after
and private equity.
the Second World War. India had a
fair development banking system and Commodity future market
was mainly targeted towards financing In the aftermath of the 2008 global financial
short–term capital requirements of the crisis, some regulatory reforms have been
industrial projects. On the contrary, DFI initiated in the US, European Union (EU)
– listed institutions like the NABARD, and some other countries to enhance market
SIDBI and NHB, were catering to the transparency and coordination among
100 Indian Financial System

regulatory authorities in the public interest, spot market. But if one wants to buy or
commodity future market is one of them. sell 15 tonne of rice at a future date, i.e.
Commodities are the primary products that after three months, one can buy or sell
can be bought, sold or traded in different the same through futures contracts at a
kinds of markets. These are the raw commodity futures exchange.
materials that are used to make secondary Let’s understand, a farmer enters into a
products which are consumed in everyday futures contract to sell 20 tonne of rice
life around the world, from food products at $ 50 per tonne to a miller on a future
to building material commodities. These are date. On that date, the miller will pay
grouped into: the full purchase price ($ 1,000) to the
i. Soft commodities: These are agricultural farmer and in exchange will receive the
products such as corn, wheat, coffee, 20 tonne of rice.
cocoa, sugar and soybean; and livestock. However, under the cash-settled
ii. Hard commodities: These are natural futures contract, the farmer and the
resources that need to be mined or miller would simply exchange the
processed such as crude oil, gold, difference between the spot price of
silver and rubber. rice on the settlement date and the
Commodity markets are of two types: agreed upon price as mentioned in the
i. Spot (physical) and ii. Derivatives (such futures contract and there would be no
as futures, options and swaps). actual delivery of rice. Following the
In a spot market, a physical commodity is above example, if on the settlement
sold or bought at a price negotiated between date the price of rice was $ 40 per
the buyer and the seller. The spot market tonne, while the agreed upon price of
involves buying and selling of commodities futures contract was $ 50 a tonne, the
in cash with immediate delivery. miller will pay $200 to the farmer in
cash and there will be no delivery of
Contracts in Derivative Markets rice to the miller. If, on the settlement
a. Forward Contract: It is a non- date, the price of rice was $ 60 a tonne,
standardized or customized contract the farmer will pay $ 200 to the miller
between two parties to undertake an in cash and no delivery of rice will take
exchange of the underlying asset at a place.
specific future date at a pre-determined c. Options Contracts: These are the
price. It is a bilateral agreement whose contracts that give the owner the right,
terms are negotiated and agreed upon but not the obligation, to buy or sell an
between two parties. It is transacted agreed amount of a commodity on or
over-the counter and is not traded on an before a specified future date.
exchange. The contract is executed by d. Swaps Contract: This contract is
both parties on the due date by delivery an agreement between two parties to
of asset by the seller and payment by exchange cash (flows) on or before
the buyer. a specified future date based on the
b. Future Contracts: These are underlying value of commodity,
agreements made on a futures currency, stock or other assets. Unlike
exchange to buy or sell a commodity at futures, swaps are not exchange-
a pre-determined price in the future. traded instruments. Swaps are usually
designed by banks and financial
For example, if one wants to buy 5
institutions that also arrange the trading
tonne of rice today, one can buy it in the of these bilateral contracts.
Indian Financial System 101

e. Over-The-Counter (OTC) derivatives: The main functions of CDE are:


These contracts are privately negotiated a. Providing and enforcing rules and
and traded between two parties, without regulations for uniform and fair trading
going through an exchange. The market practice.
players trade with one another through b. Facilitating trading in a transparent
telephone, email, and proprietary manner.
electronic trading systems. c. Recording trading transactions, including
Regulations of Commodity circulating price movements and market
Futures Markets in India news, to the participating members.
The Commodity Futures Markets in India are d. Ensuring execution of contracts.
regulated through a three-tiered regulatory e. Providing a system of protection
structure, i.e. Central Government, against default of payment (clearing).
Forward Markets Commission (FMC), and f. Providing a dispute settlement mechanism.
Commodity Exchanges. g. Designing the standardized contract
i. The Central Government: In addition for trading which cannot be modified
to determining regulatory policies, by either parties
the Central Government has the
Commodity Exchanges in India
legislative powers to pass, amend and
repeal laws related to futures trading The most important out of currently 19
in India subject to the approval of the commodity derivatives exchanges in India
Parliament. are:
ii. The Forward Markets Commission i. Multi Commodity Exchange of India
(FMC): The FMC is a statutory (MCX) – Mumbai
body set up under Forward Contracts ii. National Commodity and Derivatives
(Regulation) Act, 1952. Its headquarter Exchange of India (NCDEX) –
is at Mumbai. It is the regulatory and Mumbai
supervisory authority for commodity iii. National Multi Commodity Exchange
futures market in India. Over the (NMCE) – Ahmedabad
years, most of the regulatory powers iv. Indian Commodity Exchange (ICEX)
of the central government have been - New Delhi
delegated to FMC. It now functions v. ACE Derivatives & Commodity
under the administrative control of Exchange Limited – Mumbai
the Ministry of Finance. All terms and
vi. Universal Commodity Exchange
conditions of a futures contract have
Limited - Navi Mumbai
to be approved by the FMC before it
can be launched on commodity futures Foreign Portfolio Investment (FPI)
exchanges. It is an investment by non-residents Indians
iii. Commodity Derivatives Exchange in Indian securities, i.e. shares, government
(CDE): A commodity exchange (i.e. bonds, corporate bonds, convertible
bourse) is an organized physical or securities, infrastructure securities, etc. The
virtual marketplace where different class of investors who make investment
tradable securities, commodities in these securities are known as Foreign
and derivatives are sold and bought. Portfolio Investors.
Commodity derivatives exchanges are • SEBI has made the criteria for Foreign
places where trading of commodity Portfolio Investment. As any equity
futures and options contracts are investment by non-residents which is
conducted. less than 10% of capital in a company
102 Indian Financial System

is portfolio investment and above 10%, In other words, M4 possesses the lowest
the investment will be counted as liquidity among all these measures.
Foreign Direct Investment (FDI). The reduction in liquidity indicates the
• All FPI taken together cannot acquire shifting from ‘medium of exchange’
more than 24% of the paid up capital to ‘store of value’. All these four
of an Indian Company. money stock measures are not of equal
• Foreign Portfolio Investors are Asset importance. Their relative importance
Management Companies, Pension varies from the point of view of monetary
Funds, Mutual Funds, and Investment policy
Trusts as Nominee Companies, Generally, in developed countries, the
Incorporated / Institutional Portfolio bank deposits are the most important
Managers or their Power of Attorney
component in money supply, while due
holders, University Funds, Endowment
to less banking habits in under-developed
Foundations, Charitable Trusts and
Charitable Societies. countries people want to keep their money
in the most liquid form i.e., currency.
• Qualified Foreign Investor is an
individual, group or association which M3 is the most important component
is a resident in a foreign country. among all money stock measures and is
generally termed as ‘Broad money’.
Acronym: In economics, the money supply or
BIS = Bank for International Settlements money stock is the total amount of
CDS = Credit Default Swap monetary assets available in an economy
CFTC = Commodity Futures Trading at a specific time. There are several ways
Commission to define ‘money’ but standard measures
FII = Foreign Institutional Investor usually include currency in circulation
FMC = Forward Markets Commission
and demand deposits.
ICEX = Indian Commodity Exchange
Money supply data are usually recorded
MCA = Ministry of Corporate Affairs
MCX = Multi Commodity Exchange of and published by the government or
India the central bank of the country. Public
NAFED = National Agricultural Co- and private sector analyst have long
operative Marketing Federation monitored changes in money supply
of India Limited because of its possible effect on the price
NBOT = National Board of Trade level inflation, exchange rate and the
NCDEX = National Commodity and business cycle.
Derivatives Exchange of India
NMCE = National Multi Commodity Inflation
Exchange • As per economics concept, there might
OCEIL = Online Commodity Exchange arise a condition where there is a steep
India Limited increase in the general price of goods
UCC = Unique Client Code
and services, over a time span. During
UNCTAD = United Nations Conference on
such a crisis, a conventional currency
Trade and Development
unit buys lesser goods and services.
Money Supply Thus, there is a reduction in the
M1 measure represents the most liquid purchasing power and affordability.
form of money among four money stock Also, the currency value might dip and
measures adopted by RBI. As we move this could hit export and import and
from M1 to M4, the liquidity gets reduced. wreck and economy.
Indian Financial System 103

• The positive effects could be reduction economic growth since excess


of debt of the public and the private demand privileges investment and
sector, keeping nominal interest rates expansion.
above zero, so that the central banks (b) Cost–push inflation: Also called
can adjust interest rates to stabilize the
‘supply shock inflation.’ It is
economy and reducing unemployment.
featured by a drastic fall in the
• However, the negatives weigh more. overall output. A typical example
Inflation creates a major set – back for would be that of insurance
investment and savings, production losses during recession, due to
and promotion, thereby resulting in fraudulence or disasters.
shortage of goods and resources. In a
way, the opportunity cost of holding (c) Built–in inflation: Such
money, increases. inflation is ushered by adaptive
expectations, related to the price/
• The Consumer Price Index (CPI), the
wage spiral. Price/wage spiral is a
Personal Consumption Expenditures
humongous cyclic process, where
Price Index (PCEPI) and GDP deflator
wage increase, increases the price.
are some of the examples of broad
price indices. It could happen either because
business owners try to push profit
• Inflation does not only increase prices
margins from rising expenses or
of commodities such as food and fuels,
the wage earners try to push their
but also hike prices of financial assets
nominal after – tax wages upward
(stocks, bonds), tangible assets (such
to maintain equilibrium with the
as real estate), services (healthcare and
education) and manpower resources rising prices.
(labour). • Inflation can be checked if the economic
• For calculating the inflation rate, the growth is at par with the increase in
percentage rate of change of price the money supply. Some of the allied
index over a period of time, needs to factors are investment in market
be gauged. production, infrastructure, education,
• To study the effects of inflation, healthcare, wherein investment should
Robert Gordon’s triangle model can be hiked; even defence.
be utilized. As per the study, there are
Deflation
three types of inflation, namely:
• Deflation is a commonly witnessed
(a) Demand–pull inflation: There
economical picture when improvements
is an increment in the aggregate
in production efficiency, lower the
demand, due to an increase in
overall price of the goods. In such a
the public and private spending.
condition, the hard currency per head
A typical example is the kind of
count drops, in effect making money
disposable income which youth scarce.
today are exposed to. However, • Generally, deflation is a detriment in
demand inflation encourages the price level of goods and services.
104 Indian Financial System

A 0% (negative inflation rate) is also Indian Banking System


called deflation.
Introduction & Evolution
• Economists believe that deflation
increases the debt value, leading to
• The origin of the Indian banking system
dates back to the last decades of the 18th
recession and hence do not endorse it.
century.
• In the IS–LM Model, a fall in the
• The State Bank of India, which was
aggregate level of demand can cause initially known as the Bank of Calcutta,
deflation, due to a shift in the demand started in 1806 and was then renamed as
– supply curve for the goods and the the Imperial Bank of India, which was
services. As the prices of goods fall, partially nationalized on 1st July, 1955,
consumers tend to delay purchase and further converted into being what is
decisions which in turns hampers the called as SBI today.
overall economic activity. This condition • However, much earlier to this, the first
also negatively affects investments and officially established records as per
product innovation. Such a condition is records, was the Bank of Hindostan
called the deflationary spiral. which operated from 1770 – 1832,
• There was inflation during World War followed by the General Bank of India,
I, but deflation returned immediately which was functional from 1786 – 1791.
after the war had ended, almost close • The Indian Banking System is broadly
to the 1930s Depression. classified into Scheduled and Non –
• Ideally, deflation might also result Scheduled Banks. The banks which are
from an inequilibrium; i.e. the supply included under the 2nd Schedule of the
of goods going up and the supply of Reserve Bank of India Act, 1934, are the
money coming down. Scheduled Banks; further classified into
• Demand-based deflation are categorized nationalized banks, foreign banks and
as follows: Indian private sector banks.
(a) Growth Deflation– A deflation • In India, as nationalized banks have vast
resulting out of decrease in the coverage, they are the majority lenders
production and distribution in the Indian economy. Eg: SBI
cost of the goods and services, • The Indian Banking Act was passed in
accompanied by competitive price March 1949 and on the 1st January, of
cuts, resulting in demand rise. the same year; the Reserve Bank of India
(b) Cash Deflation– A decrease in the was nationalized.
overall consumption to save money, • In 1955, 8 other banks were converted
leads to decrease in the velocity of into SBI’s associate banks, and the State
money, otherwise termed as cash Bank Group was formed.
deflation. • The list of the 8 banks include: The State
• The most dangerous impact of deflation is Bank of Bikaner and Jaipur, The State
non–investment or reduced investment. Bank of Hyderabad, The State Bank
• Deflation can be controlled by special of Indore , The State Bank of Mysore,
liquidity policies to be strategized by The State Bank of Saurashtra, The State
the central banks, alongside regulating Bank of Patiala and The State Bank of
the value of the capital assets. Travancore.
Indian Financial System 105

the keeping of reserves with a view of


• In 1969, 14 large commercial banks,
securing monetary stability in India and
with reserves more than 50 crores each,
generally to operate the currency and
were nationalized, in order to establish
the credit system of the country to its
an authority and proper regulation. advantage.
The nationalized banks include: The
Central Bank of India, Bank of India,
• The general administration and the
direction of the RBI is governed by
Punjab National Bank, Canara Bank, a central board of directors, which is
United Commercial Bank, Syndicate nominated by the Government of India,
Bank, Bank of Baroda, United Bank with regards to the Reserve Bank of
of India, Union Bank of India, Dena India Act.
Bank, Allahabad Bank, Indian Bank, • The Central Board is appointed for a
Indian Overseas Bank and the Bank of span of 4 years and includes Official
Maharashtra Directors (1 Governor and 4 Deputy
• In 1980, 6 private banks, with reserves Governors) and Non – Official Directors
more than 200 crores each were (10 Directors and 2 Government
nationalized. Those banks were: Andhra Officials; nominated by the Government
and 4 Directors elected from the Local
Bank, Punjab and Sindh Bank, New
Boards).
Bank of India, Vijaya Bank, Corporation
Bank and Oriental Bank of Commerce. • The RBI is headquartered in Mumbai
and Dr. Urijit Patel is the Governor.
• However, as in 1993, the New Bank of
India merged with the Punjab National
• Some of the objectives include:
monitoring the issue of bank notes and
Bank, the total number of nationalized
stock – keeping reserves, for securing
banks in the country, is as of now 19. monetary stability and regulating the
• Please insert the diagram (Evolution of credit system; supervising the monetary
the Indian Banking Industry) – Pg. E – policy in India (which involves
99, General Studies, CSAT Paper – 1. formulating the SOPs, framework
and institutional composition) and
Reserve Bank of India supporting cash flow for industrial
• With a capital of 5 crores, the Reserve development; publishing legal notices
Bank of India; abbreviated as RBI, was in the cases of bank fraudulence and
established in 1935. consolidating accounts (money, debt and
foreign exchange).
• The entity originated as 5 lakh equity
shares of 100 each, with the share capital • The RBI performs a number of other
being with the non – government share developmental functions, such as
arranging credit for agriculture (now
holders.
taken over by NABARD), collecting
• In order to streamline the assets and and publishing periodic economic data,
prevent monopoly, the Reserve Bank providing loans to the Government
of India was nationalized on January 1, by buying and selling Government’s
1949. securities and trade bills.
• RBI, also known as India’s Central Bank • The RBI also represents the membership
has the following as its Preamble, “To of India in the International Monetary
regulate the issue of the Bank notes and Fund.
106 Indian Financial System

Composition of the Indian Banking System

Regional Rural Banks (RRBs) commercial policy. The Khusrau


• Even after the nationalization of banks in Committee recommended the RRB
1969, there were cultural issues, which merger with the sponsor banks.
posed as a hindrance to the government – • The RRBs are being monitored by the
led commercial banks to lend money and NABARD currently and the government
support the farmers. allowed the RRBs to grant loans to the
• To resolve the issue, the Narasimhan non – priority sectors to improve their
Working Committee was set up in 1975. financial position.
Based on this group’s recommendations,
a Regional Rural Banks Ordinance was Private Sector Banks
formulated in 1975, which was later • The Private Sector Banks are those banks
replaced by the Regional Rural Banks where the equity is held by the private
Act in 1976. shareholders and not by the government.
• The RRB’s shares were: Central Govt. • The private sector banks have been
– 50%, State Govt. – 15% and Sponsor categorized as old and new.
Bank – 35%. • The old banks existed prior to the
• The Public Sector Banks sponsor RRBs, nationalization in 1969 and were
subscribing to the share capital. trivial to be nationalized. The Board
• The RRBs cater to the credit needs of the of Directors for such banks consisted
weaker sections of the society including of eminent businessmen or prominent
farmers, artisans and small – scale personalities, mostly.
entrepreneurs. • Some of the old private sector banks
• RRBs were unable to sustain because of are City Union Bank, Dhanlaxmi Bank,
the mounting losses due to imprudent J&K Bank and the Karur Vyasa Bank
Indian Financial System 107

• The banks which initiated operations • The short – term structures include the
after the liberalization in 1991, with the State Cooperative Banks (operate at
introduction of economic and financial the state apex level), District Central
reforms, are the new private sector Cooperative Banks (operate at the district
banks. levels) and the Primary Agricultural
• Some of the criteria for being enlisted Credit Societies (operate at the village
were: banks should have minimum net level).
worth of 200 crore; within 3 years of • On the contrary, the long – term
inception, the banks should be issuing structures are bifurcated into State
shares to public and the net worth should Cooperative Agriculture and Rural
rise to 300 crore and the promoters Development Banks (SCARDS) and
should be holding 25% of the paid – up Primary Cooperative Agriculture and
capital. Rural Development Banks (PCARDS)
• These banks are further classified into – • Since these banks follow a complex
Indian and Foreign banks. Some of the operational model, a forum called the
Indian banks are Kotak Mahindra Bank, State Level Task Force on Cooperative
Yes Bank, Bandhan Bank and ICICI Union Banks (TAFCUB) has been
Bank and some of the foreign players are established to supervise and address
HDFC Bank and Deutsche Bank.
grievances related to duality of control
Cooperative Banks (between the RBI and the NABARD).
• These banks have been organized under Scheduled and Non – Scheduled
the cooperative society’s law of the Banks
states.
• The RBI has classified the banks as
• The history of this concept dates back scheduled and non – scheduled.
to the days of poverty and deprivation
in Europe, when Hermann Schulze and
• The scheduled banks are those which
have a paid – up capital and reserve an
Friedrich Wilhelm had proposed this
aggregate value of not less than 5 lakh.
idea.
• Te concept centered round the easy • The scheduled banks form the majority
availability of credit to small businesses of the banking spectrum and comprises
and for the poor sections of the society. of commercial banks (domestic and
Today, microfinance institutions follow international), RRBs and the state –
a similar approach for economic cooperative banks.
development. • For the banks which have not been
• In India, the cooperative banks have included in the Second Schedule of the
been classified into: Urban Cooperative RBI Act, 1934, they were termed as the
Banks and Rural Cooperative Banks. non – scheduled ones.
• Banking protocols of the Urban • Non – scheduled banks are also called
Cooperative Banks are monitored by Local Area Banks (LABs) are further
the RBI and the Urban Cooperatives are classified into Coastal Local Area
further bifurcated into scheduled and Banks (Andhra Pradesh), Capital Local
non – scheduled institutions. Area Banks (Punjab), Krishna Bhima
• Alternatively, the Rural Cooperatives are Samruddhi Local Area Bank Ltd
divided into short – term and long – term (Karnataka and AP) and Subhadra Local
structures. Area Bank (Kolhapur).
108 Indian Financial System

Insurance Sector • The demand for nationalization of life


insurance industry was made repeatedly
IRDA in the past but it gathered momentum
Insurance Regulatory and in 1944 when a bill to amend the Life
Insurance Act 1938 was introduced in
Development Authority (IRDA)
the Legislative Assembly.
IRDA is a multimember nodal agency and is
vested with regulatory powers in respect of • However, it was much later on the 19th
the insurance sector similar to those vested of January, 1956, that life insurance
in SEBI in respect of the capital markets in India was nationalized. About 154
and of RBI for the banking sector. The Indian insurance companies, 16 non-
IRDA Act, 1999 cleared the way for private Indian companies and 75 provident
sector entry into the insurance business. were operating in India, at the time of
nationalization.
The IRDA has powers to lay down
prudential norms and investment guidelines. • Today LIC functions with 2048 fully
It has ended the monopoly of LIC/GIC in computerized branch offices, 113
the insurance sector, by permitting private divisional offices, 8 zonal offices and
players to enter, as recommended by 1381 satellite offices. LIC’s Wide Area
Malhotra Committee. Network covers 113 divisional offices
and connects all the branches through a
Some of the functions of the authority
Metro Area Network.
include:
• LIC continues to be the dominant life
• To protect the interest of and secure fair
insurer even in the liberalized scenario
treatment to policy holders.
of Indian insurance and is moving fast
• To bring about speedy and orderly on a new growth trajectory surpassing its
growth of the insurance industry. own past records.
• To ensure speedy settlement of genuine • Some of the objectives of the LIC
claims and to prevent frauds and include increasing insurance cover;
malpractices. reaching out to the masses at reasonable
• Promote fairness, transparency and prices; maximizing savings; catering to
orderly conduct in financial markets the insurance needs of the society and
dealing with insurance. promoting satisfaction, ownership and
dedication for the agents and employees
Life Insurance Corporation of for achieving corporate targets and
India (LIC) market penetration.
• A true saga of trust – the hallmark of • Besides conventional portfolios such
LIC. as insurance plans, pension plans, unit
• The LIC was established on September plans, micro insurance plans, health
1, 1956, after nationalizing the existing plans and Aam Aadmi Bima Yojana, the
private insurance companies. LIC runs three schemes supported by
• There are 250 million LIC customers. SSF – Janashree Bima Yojana, Krishi
• The Insurance Act 1938, was the first Shramik Samajik Suraksha Yojana and
legislation governing not only life Shiksha Sahayog Yojana, for the benefit
insurance but also non-life insurance to of families below the poverty line.
provide strict state control over insurance • S. K. Roy is the Chairman of the LIC of
business. India.
Indian Financial System 109

General Insurance Corporation of Non – Banking Financial


India (GIC) Institutions (NBFIs)
• The entire general insurance business
Introduction
in India was nationalized by General
Insurance Business (Nationalization) • Non – banking financial institutions lack
Act, 1972 (GIBNA). a full banking license and cannot be
• The Government of India, in order to regulated by a national or international
streamline and benchmark the processes banking regulatory authority.
related to general insurance, took over • NBFIs supplement banks by providing
the shares of 107 companies into the the infrastructure to allocate surplus
general insurance business, leading to resources to individuals and companies
the formation of the GIC. with deficits. Additionally, NBFIs also
• The GIC was incorporated on 22nd introduces competition in the provision
November, 1972. of financial services.
• Gradually, after the mergers and • While banks have standard products and
alliances amongst the insurance players, services, NBFIs offer customized deals
four companies were left as fully owned as per client requirements.
subsidiary companies of GIC, namely • A market – oriented financial system,
National Insurance Company, United juxtaposed with well – developed NBFIs
India Insurance Company, Oriental is conducive for economic growth.
Insurance Company and New India • On the brighter side of things, NBFIs
Assurance Company.
provide additional options to transform
• GIC Re is a wholly owned subsidiary of an economy’s savings into capital
the Indian Government. investment, which serves as back – up in
• The objectives include building long case of an economic crisis.
– term relationship with the business
partners, setting ethical standards, using
• Flipside, since there is no regulatory
intervention in the case of NBFIs, they
technological intervention to reach out to
could lead to destabilizing the financial
customers with innovative products and
services and maximizing revenues. system. An illustration of the same is the
Asian Financial Crisis in 1997.
• G. C. Gaylong is the GM & Director and
GIC is headquartered in Mumbai. • According to the Economic Survey,
there are four NBFI institutions in
• GIC re has recently gone global and
enforced reinsurance programmes of India, viz. Export Import Bank of India
several insurance companies in the (EXIM), NABARD, National Housing
SAARC countries. Bank (NHB) and Small Industries
Development Bank of India (SIDBI).
• GIC provides Treaty and Facultative
business on risk emanating from the • These institutions are supervised by the
international markets, based on merits of RBI – as all – India FIs.
the business. • There is an ‘umbrella limit’ for the total
• A.M. Best has affirmed the financial resources mobilized at any point of time
strength rating of A (Excellent) and the by an FI, which should not exceed 10
issuer credit rating of “a” of General times its net owned funds as per the
Insurance Corporation of India. latest audited balance sheet.
110 Indian Financial System

Export – Import Bank of India • After RBI sold its stake in NABARD to
(EXIM) the Indian Government, the former holds
• EXIM Bank is the premier export finance 99% stake and NABARD has a corpus of
institution in India, established in 1982, more than 5000 crore.
under the Export – Import Bank of India • NABARD has been entrusted with
Act, 1981. matters concerning policy, planning
• Since its incorporation, EXIM Bank and operations in the verticals related to
has facilitated cross border trade and credit for agriculture and other economic
investments. activities in the rural areas of India.
• EXIM Bank aides the small and medium • Through assistance of Swiss Agency
for Development and Cooperation,
scale industries, through a varied
NABARD set up the Rural Innovation
portfolio of products and services, in
Fund. The Rural Infrastructure
the complete business cycle, starting
Development Fund (RIDF) is another
from import of technology, product noted scheme of 51,283 crore and have
development, marketing and export. been sanctioned for 2,44,651 projects
• EXIM Bank comprises of a Board of covering irrigation, rural roads and
Directors including representatives bridges, health and education, soil and
from the Government, RBI, Export water conservation schemes.
Credit Guarantee Corporation of India, • To provide mileage to the economic
public sector banks and the business development of the villages, NABARD
community. has process partnered with about 4000
• The main objectives of EXIM Bank is organisations in promoting SHG-Bank
financing, facilitating and promoting Linkage programme, tree-based tribal
foreign trade in India. Also, it is communities’ livelihoods initiative,
responsible for discharging duties of watershed approach in soil and
coordinating the activities of various water conservation, increasing crop
financial institutions, providing finances productivity initiatives through Lead
Crop Initiative or disseminating flow
for export and import of goods and
of information to agrarian communities
services.
through Farmer Clubs.
• EXIM is known to manage finances to
third world countries for export and National Housing Bank (NHB)
is a wholly owned subsidiary of the • National Housing Bank (NHB), a wholly
Government of India. owned subsidiary of Reserve Bank of
India (RBI), was set up on 9 July 1988
NABARD
under the National Housing Bank Act,
• NABARD was established based on 1987.
the recommendation of the Shivaram • NHB is a premier institution for housing.
Committee on July 12, 1982.
• There was a dire need for the
• NABARD is one of the premier agencies incorporation of the NHB because of the
providing economic impetus and non – availability of long – term finance
development in the rural areas. to individual households which was a
• NABARD is India’s specialised bank for major impediment to the development
Agriculture and Rural Development in of the housing sector. Thus, the NHB
India and started with an initial corpus of essentially became an Act of the
100 crore. Parliament.
Indian Financial System 111

• Some of the objectives of the NHB • The Indian Capital market does often
include integrating housing finance provide loans to the SIDBI.
system with the overall finance system, • SIDBI is also permitted to obtain loans
ensuring widespread access of housing from foreign sources through the ‘Single
facilities, promoting affordable housing Window Service.’
credit, upgrading the housing stock in
• SIDBI has contributed to the corpus
the country, encouraging public agencies of 88 venture capital funds which has
to emanate as providers of serviced catalyzed investment of more than 5600
lands, for housing and gauging optimal crore across 472 MSMEs.
utilization of resources.
• A major activity of the NHB includes Micro Finance
extending financial assistance to eligible • Microfinance is a corpus of financial
institutions in the housing sector by way services for entrepreneurs and small
of refinance and direct finance. The NHB businesses lacking access to banking and
is the regulator and supervisor of the allied services.
Housing Finance Companies (HFCs) in • Mohammed Yunus, a Nobel Prize
the country. winner, introduced the concept of
microfinance in Bangladesh in the form
Small Industries Development of the ‘Grameen Bank.’ The concept was
Bank of India (SIDBI) bought by NABARD, under the flagship
• Small Industries Development Bank microfinance initiative.
of India (SIDBI) was established as a • Microfinance accounts for a common
wholly owned subsidiary of IDBI under link between SHGs, NGOs and banks.
the Small Industries Development Bank
of India Act, 1989.
• The main attributes of microfinance
includes loans devoid of security,
• SIDBI is the apex financial institution loans to people below the poverty line,
for promoting, financing and developing regulating loan limits, assisting NGOs
industries in the small scale sector. in deciding terms and conditions for
• The operations were incorporated from granting loans to the poor and exploiting
April 2, 1990. untapped business potential of the rural
• Headquartered in Lucknow, SIDBI areas.
regulates the activities of agencies which • Launched in 1992 as a pilot project,
provide funds to small enterprises. it has since provided its efficacy as a
• The entire portfolio of operations which mainstream programme for banking
were previously handled by the IDBI has by the poor, who mainly comprise the
been shifted to SIDBI. marginal farmers, landless labourers,
• SIDBI provides assistance to the small artisans and craftsmen and people
scale industrial sector through additional involved in hawking and vending
institutions like State Financial business.
Corporation (SFC), Commercial Banks • Microfinance is advantageous as it
and State Industrial Development ensures timely repayment of loans to
Corporation. banks, reduction in transactional costs,
• Besides share capital, SIDBI can both to the poor and the banks and
increase resources by taking loans from maintaining transparency and coherence
the Government of India and RBI. in financial proceedings.
112 Indian Financial System

• There are currently a few social Pvt. Ltd., Janalakshmi Financial Services
interventions that have been combined Pvt. Ltd., S.M.I.L.E Microfinance Ltd.
with micro financing to increase and Utkarsh Microfinance Ltd.
awareness of HIV/AIDS. Such initiatives FOREIGN BANKS
like the “Intervention with Microfinance
for AIDS and Gender Equity” (IMAGE) • The Mandate of the Department of
Financial Services covers the functioning
which incorporates microfinance with
of Banks, Financial Institutions,
“The Sisters-for-Life” program; a
Insurance Companies, Foreign Banks
participatory program that educates on and the National Pension System.
different gender roles, gender-based
violence, and HIV/AIDS infections to
• The Department is headed by the
Secretary, who is assisted by an
strengthen the communication skills and
Additional Secretary, four Joint
leadership of women. Secretaries and two Economic Advisers.
• Few of the most reputed players are SKS • The following tabular column represents
Microfinance Ltd., BSS Microfinance some of the most reputed foreign banks,
Pvt. Ltd., Grameen Financial Services operating in India:

Bank Name Country Number of Branches

Standard Chartered Bank UK 81


HSBC Hong Kong 45
Citibank USA 39
ABN – Amro Bank Netherlands 24
BNP Paribas France 8
Deutsche Bank Germany 8
Bank of Nova Scotia Canada 5
Bank of America USA 5
Bank of Bahrain & Kuwait Bahrain 2
Barclays Bank UK 1

• Not only do these foreign banks of them have been able to make a mark
provide innovative financial services to in this segment of the banking industry.
customers, but also create an all new In an era when retail sector accounts for
banking experience, using the power of 20% of all the credits, the disappearance
technology. of foreign sector banks could greatly
• Foreign banks also play a significant role undermine the competitiveness and
in shaping up a country’s economy and efficiency of the Indian Retail Banking
is a relationship – building tool among Industry.
nations, considering the concepts of
macroeconomics.
• The RBI is primarily responsible for
monitoring the regulatory framework
• Though foreign banks bring with them all
with respect to the foreign banks.
the technologies and expertise, yet none
Indian Financial System 113

1. What are the three important components Which of the above statements are
of the Indian Money Market? TRUE?
(a) Liquid Money, White Money and (a) (i) and (ii)
Black Money (b) (ii) and (iii)
(b) Short – term money and Long – (c) (i) and (iii)
term money (d) (i), (ii) and (iii)
(c) Call Money, Notice Money and 6. For which of the following amounts,
Term Money can T – Bills be issued?
(d) Liquid Money, Notice Money
(a) ` 75, 000
and Term Money
(b) ` 70, 000
2. Consider the following statements with
(c) ` 60, 000
respect to the Indian Money Markets
(i) In India, the privatization accounts (d) ` 45, 000
for catering to the majority of the 7. Which is the most significant utility of
financial demands the commercial bill market?
(ii) Inter – Bank Market involves (a) Timely collection of interests
transactions between banks for (b) Short – term funds for business
creating financial equilibrium empowerment
(iii) Most of these exchanges take (c) Create a stable seller – buyer
place considering floating interest relationship
rates. (d) Identify payment trespassers.
Which of the above statements are 8. What is the biggest challenge for the
TRUE?
commercial bill?
(a) (i) & (ii) (b) (ii) & (iii)
(c) (i) & (iii) (d) (i), (ii) & (iii) (a) Big business conglomerates
3. What is T – Bills? (b) Private banks
(a) Telephone Bills (c) Consumers
(b) Telemarketing Bills (d) Sellers.
(c) Tenure Bills 9. Consider the statements related to the
(d) Treasury Bills Collateral Loan Market
4. Which country’s government used T – (i) There is a huge risk involved in
Bills for the first time? the high value loans
(a) USA (ii) Collateral Loan Market facilitates
(b) USSR loan sanction real fast
(c) UK (iii) Maturities on such loans are no
(d) India longer than one year.
5. Which of the following aspects are
Which of the above statements are
unique to the USP of the T – Bills?
TRUE?
(i) An auction process
(a) (i) and (ii)
(ii) Competitive returns as per
customers’ specifications (b) (ii) and (iii)
(iii) Government bears the (c) (i) and (iii)
responsibility of the final payment. (d) (i), (ii) and (iii)
114 Indian Financial System

10. Who are the lenders in the case of (c) when the exchange rate of a
collateral loans? currency falls
(a) Asset Management Companies (d) when fiscal deficit exceeds
(b) Banks balance of payment deficit
(c) Insurance Companies 16. According to the Chakravarthy
(d) Credit Rating Agencies Committee, one of the principal causes
11. Which financial enterprise assists in affecting price stability in India is:
covering short – term obligations like (a) existence of black money
payroll? (b) violent fluctuation in agricultural
(a) Commercial Bill Market production
(b) Collateral Loan Market (c) India’s precarious balance of
(c) Treasury Bill Market payment position
(d) Commercial Paper Market (d) fiscal deficit
12. Consider the following attributes of 17. Which of the following statements
correctly expresses the difference
gilt – edged securities.
between preference shares and equity
(i) Safe investment
shares?
(ii) Complete liquidity
(a) equity shareholders have no
(iii) Bulk investment
voting right but preference
Which of the above is TRUE?
shareholders have voting rights
(a) (i) and (ii)
(b) preference shareholders have
(b) (i) and (iii) no have voting rights but equity
(c) (ii) and (iii) shareholders have voting rights
(d) (i), (ii) and (iii) (c) preference shareholders have
13. The first public sector bank in India no right to profit whereas equity
which obtained license for Internet shareholders have a right to profit
Banking from RBI is- (d) preference shareholders get
(a) Punjab National Bank exemption from taxes while
(b) Oriental Bank of Commerce equity shareholders do not get
(c) Corporation Bank any exemption
(d) State Bank of India 18. Which one of the following
14. Which of the following is not a function governmental steps has proved
of the Securities and Exchange Board relatively effective in controlling the
of India (SEBI)? double digit rate of inflation in the
(a) Supervising the working of the Indian economy during recent years?
Stock Exchanges (a) enhanced rate of production of all
(b) Underwriting new capital issues consumer goods
(c) Regulating merchant banks and (b) streamlined public distribution
mutual funds system
(d) promoting the development of a (c) pursuing an export oriented
healthy capital market strategy
15. Inflation, in theory, occurs: (d) containing budgetary deficit and
(a) when the price of essential unproductive expenditure
commodities outstrips income 19. Which one of the following is not a
(b) when money supply grows at feature of “Value Added Tax” ?
a higher rate than GDP in real (a) It is multi-point destination-
terms based system of taxation.
Indian Financial System 115

(b) It is a tax levied on value addition (b) a rise in prices of shares of


at each stage of transaction in the all companies registered with
production distribution chain. National Stock Exchange
(c) It is a tax on the final consumption (c) an overall rise in prices of shares
of goods or services and must of group up companies registered
ultimately be borne by the with Bombay Stock Exchange
consumer. (d) a rise in prices of shares of all
(d) It is basically a subject of the companies belonging to a group
central government and the state of companies registered with
governments are only a facilitator Bombay Stock Exchange
for its successful implementation. 23. Which of the following could be a
20. Which one of the following statements cause of demand inflation?
is an appropriate description of (a) An increase in the cost of labour
deflation? (b) An increase in domestic interest
(a) It is a sudden fall in the value of a rates
currency against other currencies (c) An increase in the level of
(b) It is a persistent recession in both consumer spending
the financial and real sectors of (d) An increase in import prices,
resulting from a depreciating dollar
economy
24. ‘SHG Bank Linkage Programme’
(c) It is a persistent fall in the general
is a programme which encourages
price level of goods and services
India’s banks to lend to self-help
(d) It is a fall in the rate of inflation
groups (SHGs) composed mainly of
over a period of time
poor women, this has evolved into an
21. How do we define the terms bull and
important Indian tool for microfinance.
bear with regard to stock markets? This programme was initiated by ?
(a) A bull is an optimistic operator (a) Reserve Bank of India (RBI)
who first buys and then sells (b) Agricultural Refinance and
shares in expectation of the price Development Corporation
going up; a bear is a pessimistic (ARDC)
market operator who sells the (c) National Bank for Agriculture and
shares in expectation of buying Rural Development (NABARD)
them back at a lower price. (d) Non-Banking Finance
(b) There is nothing significantly Companies (NBFC)
different as both operate in the 25. Which of the following statements
capital market. is not correct regarding the ‘Banking
(c) Bull is one who first sells a share Sector’ of India?
and then buys it at a lower price; (a) At present there are 26
bear means one who first buys Nationalized Banks in India.
and then sells it in expectation of (b) Foreign Banks and Regional Rural
prices going up. Banks do not come under the
(d) A bull is ready to buy any share; category of Scheduled Commercial
a bear only deals in government Banks.
securities. (c) Banks have the freedom to
22. A rise in ‘SENSEX’ means: regulate their own Savings Bank
(a) a rise in prices of shares of Deposit interest rates.
all companies registered with (d) Narsimham Committee is related
Bombay Stock Exchange to Banking Sector reforms.
116 Indian Financial System

26. The concept of ‘Universal Banking’ 31. Which among the following would
was implemented in India on the most likely follow if the Reserve
recommendations of: Bank of India effects selling of the
(a) Abid Hussain Committee securities?
(b) R H Khan Committee (a) The cash resources at the disposal
(c) S Padmanabhan Committee of the commercial banks increase.
(d) Y H Malegam Committee (b) The cash resources at the disposal
27. When there is an inflationary trend in of the commercial banks get
the economy, what would be trend in diminished.
the pricing of the Bank Products? (c) The cash resources of the
(a) Increasing Trend commercial banks remain
(b) Decreasing Trend
unchanged.
(c) Constant Trend
(d) None of the above.
(d) There is no relevance of the
32. An excise duty is a charge on which of
inflation in pricing of the Banking
the following?
Products
(a) Export of goods
28. As per Section 24 (2A) of Banking
Regulation Act 1949, every banking (b) Production of goods
company in India has to maintain (c) Production or sale of goods
equivalent to an amount which shall (d) Consumption of good
not at the close of the business on 33. The most active segment of the Money
__________ be less than 25% of Market in India is which one of the
the total of its net demand and time following?
liabilities, which is known as SLR. (a) Call Money / Notice Money
Which among the following is the Market
correct option? (b) Repo / Reverse Repo
(a) Any Day (c) Commercial Paper (CP)
(b) Any Week (d) Certificate of Deposit (CD)
(c) Any Fortnight 34. What does venture capital mean?
(d) Any Month (a) A short-term capital provided to
29. For which of the following, the industries
Reserve Bank of India has stipulated (b) A long-term start-up capital
a maximum Capital Adequacy provided to new entrepreneurs
Requirements in India? (c) Funds provided to industries at
(a) Private Sector Banks times of incurring losses
(b) Banks that Undertake Insurance (d) Funds provided for replacement
Business. and renovation of industries
(c) Local Area Banks
35. Consider the following statements :
(d) Scheduled Commercial Banks
The function of the Reserve Bank of
30. Which one of the following Public
India does not include:
Sector Bank’s emblem figures a dog
1. Open market operations
and the words ‘faithful friendly, in it?
(a) Punjab National Bank 2. Monitoring revenue collection
(b) Syndicate Bank 3. Supervising non-banking finance
(c) Oriental Bank of Commerce companies
(d) State Bank of India 4. Review of public expenditure
Indian Financial System 117

Which of the statement/s given above 39. Which of the following measures
is/are correct? would result in an increase in the
(a) 1 only (b) 2 and 4 money supply in the economy?
(c) 1, 3 and 4 (d) 1, 2, 3 and 4 1. Purchase of government securities
36. Consider the following statements. from the public by the Central
1. The National Housing Bank, Bank
the apex institution of housing 2. Deposit of currency in
finance in India, was set up as a commercial banks by the public
wholly-owned subsidiary of the 3. Borrowing by the government
Reserve Bank of India. from the Central Bank
2. The Small Industries 4. Sale of government securities to
Development Bank of India was the public by the Central Bank
established as a wholly-owned Select the correct answer using the
subsidiary of the Industrial codes given below :
Development Bank of India. (a) 1 only (b) 2 and 4
Which of the statement given above is/ (c) 1 and 3 (d) 2, 3 and 4
are correct? 40. Consider the following statements:
(a) 1 only (b) 2 only 1. High growth will lead to inflation.
(c) Both 1 and 2 (d) Neither 1 nor 2 2. High growth will lead to
37. Why is the offering of “teaser loans’’ deflation.
by commercial banks a cause of Which of the statements given above
economic concern ? is/are correct?
(a) Only 1 (b) Only 2
1. The teaser loans are considered to
(c) Both 1 and 2 (d) Neither 1 nor 2
be an aspect of sub-prime lending
41. Which one among the following is an
and banks may be exposed to the
appropriate description of deflation?
risk of defaulters in future.
(a) it is a sudden fall in the value of a
2. In India, the teaser loans are
currency against other currencies
mostly given to inexperienced
(b) It is a persistent recession in the
entrepreneurs to set up
economy
manufacturing or export units.
(c) It is a persistent fall in the general
Which of the statements given above
price level of goods and services
is/are correct?
(d) It is fall in the rate of inflation
(a) 1 only over a period of time
(b) 2 only 42. Consider the following statements
(c) Both 1 and 2 regarding Non-Banking Finance
(d) Neither 1 nor 2 Companies (NBFCs):
38. Name the 3 commodities in the 1. NBFCs can also engage in Micro-
Wholesale Price Index. Finance Activities.
1. Primary articles 2. Housing-finance companies
2. Fuel, power, light and lubricants form a distinct sub-group of the
3. Manufactured products NBFCs.
4. Food articles and industrial raw 3. The deposit insurance facility of
materials the Deposit Insurance and Credit
(a) 1, 2 and 3 (b) 2, 3 and 4 Guarantee Corporation is not
(c) 1, 3 and 4 (d) 1 and 3 available for NBFC depositors.
118 Indian Financial System

Which among the above statement(s) 47. With reference to inflation in India,
is/are not correct? which of the following statements is
(a) Only 1 (b) 1 and 3 correct?
(c) All of there (d) None of these (a) Controlling the inflation in
43. Which among the following are the India is the responsibility of the
‘Credit-Rating Agencies’ of India? Government of India only
1. CRISIL 2. CARE (b) The Reserve Bank of India has no
3. ICRA 4. ONICRA role in controlling the inflation
(a) 1 and 3 (b) 1, 2 and 3 (c) Decreased money circulation
(c) 1, 3 and 4 (d) All of them helps in controlling the inflation
44. Which among the following are the (d) Increased money circulation
wholly/partly owned subsidiaries of helps in controlling the inflation
the Reserve Bank of India (RBI)? 48. The Reserve Bank of India regulates
1. Deposit Insurance and Credit the commercial banks in matters of:
Guarantee Corporation (DICGC) 1. liquidity of assets
2. National Housing Bank (NHB) 2. branch expansion
3. National Bank for Agriculture and 3. merger of banks
Rural Development (NABARD) 4. winding-up of banks
4. Bharatiya Reserve Bank Select the correct answer using the
Note Mudran Private Limited codes given below.
(BRBNMPL)
(a) 1 and 4 only
(a) 1 and 4 (b) 1, 2 and 4
(b) 2, 3 and 4 only
(c) 2, 3 and 4 (d) All of them
(c) 1, 2 and 3 only
45. Consider the following steps:
(d) 1, 2, 3 and 4
1. Banking Department gets
49. Which of the following grants /
currency from the Issue
grant direct credit assistance to rural
Department
households?
2. Government puts the currency in
1. Regional Rural Banks
circulation
2. National Bank for Agriculture
3. Central Government incurs a
and Rural Development
deficit in its Budget
4. Government Sells Treasury Bills 3. Land Development Banks
to Banking Department of RBI Select the correct answer using the
Which among the following is the codes given below.
correct order of the above steps? (a) 1 and 2 only (b) 2 only
(a) 1 2 3 4 (b) 2 3 4 1 (c) 1 and 3 only (d) 1, 2 and 3
(c) 3 4 1 2 (d) 4 3 2 1 50. Which among the followings is the
46. Consider the following: type of inflation?
1. Commercial Banks 1. Demand Pull Inflation
2. Central Bank of India 2. Cost Push Inflation
3. Government 3. Stagflation
Which among the above can create 4. Hyperinflation
money? Choose the correct type.
(a) Only 1 (b) Only 1 & 2 (a) 1, 2, 3, 4 (b) 1, 2
(c) Only 3 (d) All 1 2 & 3 (c) 3, 4 (d) 1, 4
Indian Financial System 119

51. What is the difference between Inflation 54. Which among the following are the
and Deflation? Nationalised Banks?
1. Inflation is an increase in price 1. Bank of India
of goods while Deflation is that 2. Punjab National Bank
state in which the value of money 3. Canara Bank
rises and the price of goods and 4. United Commercial Bank
services falls. Choose the code.
2. Deflation is an increase in price (a) 1 and 2 (b) 3 and 4
of goods while Inflation is that (c) 1 and 4 (d) 1, 2, 3, 4
state in which the value of money 55. What are the reforms of Narsimham
rises and the price of goods and Committee on Banking sector?
services falls. 1. Nationalisation of banks should
3. Inflation is that state in which not take place any more.
the value of money rises and the 2. Private and foreign banks should
price of goods and services falls be set up to promote competition.
while deflation is an increase in 3. There should be a phased
price of goods. reduction of CRR and SLR.
Choose the correct difference between Choose the correct option.
Inflation and Deflation. (a) 1 only
(a) 1 only (b) 2 only (b) 2 only
(c) 3 only (d) 1, 2, 3 (c) 3 only
52. Which among the following is the type (d) All of the Above
of organised sector of Indian money 56. Consider the following statement:
market? 1. As per recommendations of
1. Call money market the Narasimham Committee,
2. Treasury Bill Market it has been decided that credit
3. Commercial Bill Market facilities granted by banks will
4. Collateral loan market be classified into performing and
Choose the correct code. non-performing assets (NPA)
(a) 1 and 2 2. NPA is a loan which is in default
(b) 3 and 4 for more than nine months.
(c) 1, 2, 3, 4 Choose the incorrect statement.
(d) None of the Above (a) 1 only
53. Consider the following statement: (b) 2 only
1. Ad hoc treasury bills are sold (c) 1 and 2
to the banks and public and are (d) None of the Above
freely marketable. 57. Choose the type of capital market:
2. Regular treasury bills are not 1. Securities Market
sold to the banks and the general 2. Financial institutions
public, and are not marketable. 3. Gill-edged market
Choose the correct code. 4. Incorporated securities
(a) 1 only Which among the following code is
(b) 2 only correct?
(c) 1 and 2 (a) 1 and 4 (b) 1 and 3
(d) None of the Above (c) 1 and 2 (d) 3 and 4
120 Indian Financial System

58. Consider the following statement: 61. Which of the following markets are
1. GIC was formed in November independently regulated by Forward
1972. Market Commission?
2. The 107 private companies (a) Mutual Funds
operating in the field were (b) Commodity Futures Market
grouped together into four - (c) Stock Market
National Insurance Company, (d) Foreign Exchange Markets
United India Insurance Company, 62. According to the Chakravarthy
Oriental Insurance Company and Committee, one of the principal causes
New India Assurance Company. affecting price stability in India is:
Choose the incorrect statement. (a) existence of black money
(a) 1 only (b) 2 only (b) violent fluctuation in agricultural
(c) 1 and 2 production
(d) None of the Above (c) India’s precarious balance of
59. Name of four institutions that are payment position
regulated by the RBI as all-India FIs: (d) fiscal deficit
1. Export Import Bank of India 63. Which one of the following statements
(EXIM Bank) is an appropriate description of
2. National Bank for Agriculture and deflation?
Rural Development (NABARD) (a) It is a sudden fall in the value of a
3. National Housing Bank (NHB) currency against other currencies
4. Small Industries Development (b) It is a persistent recession in both
Bank of India (SIDBI) the financial and real sectors of
Choose the correct answer. economy
(a) 1 and 2 (c) It is a persistent fall in the general
price level of goods and services
(b) 3 and 4
(d) It is a fall in the rate of inflation
(c) 1, 3, 4
over a period of time
(d) 1, 2, 3, 4 64. The International Development
60. Consider the following statement: Association, a lending agency, is
1. IRDA is a multimember nodal administered by the:
agency (a) International Bank for
2. It is vested with regulatory powers Reconstruction and Development
in respect of the insurance sector (b) International Fund for
similar to those vested in SEBI in Agricultural Development
respect of the capital markets and (c) United Nations Development
of RBI for the banking sector. Programme.
(d) United Nations Industrial
3. The IRDA Act, 1999 cleared the
Development Organization
way for private sector entry into
65. A fall in ‘SENSEX’ means:
the insurance business. (a) a fall in prices of shares of
Choose the correct statement. all companies registered with
(a) 1 only Bombay Stock Exchange
(b) 2 only (b) a fall in prices of shares of
(c) 3 only all companies registered with
(d) All of the Above National Stock Exchange
Indian Financial System 121

(c) an overall fall in prices of shares 70. Consider the following statements:
of group up companies registered 1. Sensex is based on 50 of the
with Bombay Stock Exchange most important stocks available on the
(d) a fall in prices of shares of all Bombay stock Exchange (BSE).
companies belonging to a group 2. For calculating the Sensex, all
of companies registered with the stock are assigned proportional
Bombay Stock Exchange weightage.
66. From time to time, which among 3. New York Stock Exchange is the
the following body publishes the oldest stock exchange in the world.
“Exchange Control Manual” in context Which of the statements given above
with the Foreign Exchange in India? is/are correct?
(a) Foreign Trade Promotion Board (a) 2 only (b) 1 and 3
(b) Department of Commerce (c) 2 and 3 (d) none
(c) Reserve Bank of India 71. Brent index is associated with :
(d) SEBI (1) crude oil prices
67. The Laffer curve is the graphical (2) copper future prices
representation of: (3) gold future prices
(a) The relationship between tax
(4) shipping rate index
rates and absolute revenue these
Which of the following is true?
rates generate for the government.
(a) Only 1 (b) Only 2
(b) The inverse relationship between
(c) Only 3 (d) Only 4
the rate of unemployment and the
72. Which of the following are the
rate of inflation in an economy.
provisions of SARFAESI Act which
(c) The inequality in income
enables banks to reduce their non-
distribution.
(d) The relationship between performing assets (NPAs)?
environmental quality and 1. Enforcement of Security Interest
economic development. by secured creditor (Banks/
68. Which among the following is true Financial Institutions).
about “deficit financing”? 2. Transfer of non- performing
(a) Public expenditure in excess of assets to asset reconstruction
public revenue company which will then dispose
(b) Public revenue in excess of of those assets and realise the
public expenditure proceeds.
(c) New currency replaced by old 3. To provide a legal framework for
currency securitization of assets.
(d) None of above 4. Assisting banks in making
69. What do we call an arrangement the credibility track record
whereby an issuing Bank at the request of customers under Credit
of the Importer (Buyer) undertakes Information Bureau of India
to make payment to the exporter (CIBIL).
(Beneficiary) against stipulated (a) 1 and 2 (b) 1, 2 and 3
documents? (c) 2, 3 and 4 (d) 1, 2, 3 and 4
(a) Bill of Exchange 73. Which among the following are the
(b) Letter of Exchange ‘Credit-Rating Agencies’ of India?
(c) Letter of Credit 1. CRISIL 2. CARE
(d) Bill of Entry 3. ICRA 4. ONICRA
122 Indian Financial System

(a) 1 and 3 (b) 1, 2 and 3 Choose the correct answer.


(c) 1, 3 and 4 (d) All of them (a) 1 and 2 (b) 3 and 4
74. Choose the type of capital market: (c) 1, 3, 4 (d) 1, 2, 3, 4
1. Securities Market 78. Which of the following organisation
2. Financial institutions provides guarantee to exporters?
3. Gill-edged market (a) EXIM Bank
4. Incorporated securities (b) Export Loan Guarantee
Which among the following code is Corporation
correct? (c) RBI
(a) 1 and 4 (b) 1 and 3 (d) Commerce Ministry
(c) 1 and 2 (d) 3 and 4 79. Which of the following is not one of
75. Consider the following statement: the features of the Special Economic
1. Bombay Stock Exchange (BSE) Zones (SEZ) being set up for
is India’s oldest stock exchange promoting exports ?
2. It formally came into being in (a) The SEZ area will be treated
1888 as foreign territory for trade
3. It was a regional exchange till operations, duties and tariff.
2002 when it became a national (b) No licence is required for import
exchange into the zone.
Choose the incorrect statement (c) Foreign workers will be
regarding BSE. allowed free entry without visa
(a) 1 only
restrictions.
(b) 2 only
(d) There will be no routine
(c) 3 only
examination by customs authorities
(d) None of the Above
of import/export cargo.
76. _______ deals with long-term finance
80. TRIPS (Trade Related aspects
(more than 365 days) funds. It
of Intellectual Property Rights)
includes all facilities and institutional
agreements is administered by
arrangements available for borrowing
(a) United Nations Conference
and lending of term funds (including
on Trade and Development
medium-term).
(UNCTAD)
1. Capital market
(b) United Nations Organization
2. Stock market
3. Debit (UNO)
4. Credit (c) World Trade Organization
Choose the correct answer. (WTO)
(a) 1 only (b) 2 only (d) World Bank (WB)
(c) 3 only (d) 4 only 81. Foreign Direct Investment involves:
77. Name of four institutions that are (a) A speculator trying to make a
regulated by the RBI as all-India FIs: profit by buying company shares
1. Export Import Bank of India on a foreign stock exchange.
(EXIM Bank) (b) A UK energy company buying
2. National Bank for Agriculture and territory abroad where it expects
Rural Development (NABARD) to find oil reserves.
3. National Housing Bank (NHB) (c) A tourist purchasing foreign
4. Small Industries Development currency to spend on a holiday
Bank of India (SIDBI) abroad.
Indian Financial System 123

(d) A company signing an agreement II. These schemes (MEIS and


with a wholesaler to distribute its SEIS) replace multiple schemes
products in foreign markets. earlier in place, each with different
82. What is meant by term Balance of conditions for eligibility and usage.
Payment? III. Incentives (MEIS & SEIS) to be
I. Those transactions arising out of available for SEZs also e-Commerce
exports and imports (the visible of handicrafts, handlooms, books,
items) etc. eligible for benefits of MEIS.
II. It is astatistical statement of all IV. FTP benefits from both MEIS &
transactions made between one SEIS will be extended to units
particular country and all other
located in SEZs.
countries during a specified
(a) I & II (b) II & IV
period of time
(c) Only III (d) All the above
III. This account is the summary of all
86. Which points are correct regarding
international trade transactions of
the domestic country in one year SEZ act?
(a) I & III (b) Only I I. Exemption to SEZ developer and
(c) Only II (d) None of the units from Minimum Alternate
above Tax.
83. Which of the below statements are II. Constitution of an authority for
correct regarding exchange trade? each SEZ with a view to providing
I. The value of rupee was managed greater administrative, financial
by the state bank and functional autonomy to these
II. The strict foreign exchange zones.
controls also encouraged hawala III. Establishment of designated
trade courts and a single enforcement
III. India followed a strongly inward agency to ensure speedy trial
looking policy, laying stress on and investigation of offences
import substitution committed in SEZs.
(a) Only I (b) Both II & III (a) I & II (b) Only II
(c) Only III (d) All the above (c) Only III (d) All the above
84. What was the main reason of 87. Which among the following is the apex
introducing trade reforms in 1991? organization of Industrial Finance in
I. Make exports competitive
India?
II. Unshackle foreign trade from the
(a) IDBI (b) ICICI
clutches of a control regime
(c) IFCI (d) RIDF
III. Allow import of most goods
88. Which among the following is the
using only tariff as a restraint
(a) I & II (b) Only II correct full form of SIDO?
(c) Only III (d) All the above (a) Small Industries Development
85. Which is the correct statement Organization
according to FTP 15 – 20? (b) Sick Industries Development
I. FTP 2015-20 introduces two new Organization
schemes, namely “Merchandise (c) Small Industries Development
Exports from India Scheme Office
(MEIS)” and “Services Exports (d) State Industrial Development
from India Scheme (SEIS)” Organization
124 Indian Financial System

89. With reference to the National Which of the statements given above
Investment Fund to which the is/are correct ?
disinvestment proceeds are routed, (a) 1 and 2 (b) 2 only
consider the following statements ? (c) 3 and 4 (d) 3 only
1. The assets in the National 90. SEZs were established with the
Investment Fund are managed by objective of ____ .
the Union Ministry of Finance. 1. attracting foreign investment
2. The National Investment Fund directly.
is to be maintained within the 2. protect domestic market from
Consolidated Fund of India. competition from multinationals.
3. Certain Asset Management 3. providing more capital to
companies are appointed as the agricultural and allied activities.
fund managers. (a) 1 only
4. A certain proportion of annual (b) 2 only
income is used for financing (c) 3 only
select social sectors. (d) All of the above
Indian Financial System 125

1. (c) The three important components 10. (b) Banks are the lenders. In case of
of the Indian Money Market payment default, the property is
are call money (one – day seized by the hypothecary bank.
transactions), notice money 11. (d) Commercial paper market
(transaction between 2 – 14 days) provides a means for corporations
and term money (transactions to borrow money to cover short-
exceeding a fortnight). term debt obligations, such as
2. (b) Options ii and iii are correct payroll.
as Inter – bank market implies 12. (d) Gilt–edged securities provide
transaction (borrowing and zero risk, 100% liquidity and
lending) between banks and all massive returns.
these policies deploy floating 13. (a) 14. (b) 15. (b)
rates of interest, as they are
subjected to market risks. 16. (b) 17. (c) 18. (d)
3. (d) T – bills stands as an acronym for 19. (d) VAT is the State Subject.
treasury bills. 20. (c) Deflation is a decrease in the
4. (a) USA had introduced Treasury prices of goods and services. It
Bills for the first time in the occurs when the annual inflation
history of short – term financial rate falls below 0%, which is
markets. negative inflation rate. This is
5. (d) All three options are appropriate different from Disinflation which
as T – Bills are issued by a is a slow-down in the inflation
bidding process; for competitive rate. This is a situation when
T – Bills, the bidder has to specify inflation declines to lower levels
returns and in case of maturity, the but prices continue to rise.
government pays the par value to
21. (a) 22. (c) 23. (c)
the bearer.
24. (c)
6. (a) T – Bills can be issued for a
minimum amount of ` 25, 000 25. (b) Foreign Banks and Regional
and in multiples of the same. Rural Banks also come under
Thus option A suffices. the category of Scheduled
7. (b) A commercial bill provides Commercial Banks. Now, the
short – term boost for business banks are free to determine their
expansion savings bank deposit interest
8. (a) The MNCs and big corporate rate, subject to the following two
do not abide by the principle conditions:
of timely payment and hence 1. Each bank will have to offer
challenges the conventional a uniform interest rate on
protocol of the commercial bill. savings bank deposits up to
9. (a) The third statement is wrong as ` 1 lakh,irrespective of the
maturities on collateral loans are amount in the account within
no longer than nine months. this limit.
126 Indian Financial System

2. For savings bank deposits of inflation. It means the value


over ` 1 lakh, a bank may of money increases rather than
provide differential rates decreases.
of interest, if it so chooses. 42. (d) 43.  (d) 44.   (d)
However, there should not
45. (c) 46.  (d)
be any discrimination from
customer to customer on 47. (c) When inflation becomes very
interest rates for similar high, the RBI decreases supply
amount of deposit. of money (to check inflation) by
adopting light monetary policy.
26. (b) 27. (a) 28. (a)
48. (d) The Reserve Bank of India is the
29. (c)
main monetary authority of the
30. (b) Syndicate Bank. The symbol country and beside that, in its
of dog implies that Bank is capacity as the central bank, acts
trustworthy and a friend. Its as the bank of the national and
slogan is : Your faithful and state governments. Sometimes it
friendly financial partner. happens that some of the banks
31. (b)   32. (c)   33.   (a) close down due to non recovery
34. (b) Venture capital (VC) is a long of loans or such other issues. In
term financial capital provided to such conditions people have to
early-stage, high-potential, start suffer as their money is with the
up companies or new companies. bank then. For this reason there
is provision for winding up of
35. (b) 36.  (c)
the banking company under the
37. (a) The statement (1) is correct Banking Regulation Act, 1949.
because it includes the definition The power of winding up of Bank
of teaser loans but the statement lies in the hand of Reserve Bank
(2) is not correct, because in India of India.
teaser loan is provided to the
49. (c) Land development bank started
home buyers not for setting up
financing long term loan for more
manufacturing or export units.
significant rural development
38. (a) 39.  (c) activities like rural and cottage
40. (a) Typically, higher inflation is industries, rural artisans etc.
caused by strong economic The main purpose of RRBs is
growth. If Aggregate demand to mobilize financial resources
in an economy expanded faster from rural / semi-urban areas
than aggregate supply, we would and grant loans and advances
expect to see a higher inflation mostly to small and marginal
rate. If demand is rising faster farmers, agricultural labourers
than supply, then this suggests and rural artisans.
that economic growth is higher 50. (a) Types of inflation are Demand
than the long run sustainable rate Pull Inflation, Cost Push Inflation,
of growth. Stagflation, Hyperinflation.
41. (c) Deflation is defined as a fall in 51. (a) Inflation is an increase in price
the general price level of goods of goods while Deflation is that
and services. It is a negative rate
Indian Financial System 127

state in which the value of money the holding company.


rises and the price of goods and 59. (d) According to the Economic
services falls. Survey, there are four Institutions,
52. (c) The type of organised sector of namely the Export Import Bank
Indian money market are Call of India (EXIM Bank), National
Money Market, Treasury Bill Bank for Agriculture and Rural
Market, Commercial Bill Market Development (NABARD),
and Collateral loan market
the National Housing Bank
53. (d) Ad hoc treasury bills are not (NHB) and Small Industries
sold to the banks and the general Development Bank of India
public, and are not marketable
(SIDBI).
while regular treasury bills are
sold by Reserve Bank of India on 60. (d) IRDA is a multimember
behalf of the Central Government. nodal agency. It is vested with
54. (d) The nationalised banks are regulatory powers in respect of
Central Bank of India, Bank of the insurance sector similar to
India, Punjab National Bank, those vested in SEBI in respect of
Canara Bank, United Commercial the capital markets and of RBI for
Bank, Syndicate Bank, Bank of the banking sector.
Baroda, United Bank of India, The IRDA Act, 1999 cleared the
Union Bank of India, Dena Bank, way for private sector entry into
Allahabad Bank, Indian Bank, the insurance business.
Indian Overseas Bank Bank of 61. (b) 62. (b)
Maharashtra.
63. (c) Deflation is a decrease in the
55. (d) prices of goods and services. It
56. (b) NPA is a loan (whether term loan, occurs when the annual inflation
cash credit, overdraft, or bills rate falls below 0%, which is
discounted), which is in default negative inflation rate. This is
for more than six months. different from Disinflation which
57. (c) The securities market is further is a slow-down in the inflation
divided into the gilt-edged market rate. This is a situation when
and the corporate securities inflation declines to lower levels
market. but prices continue to rise.
58. (d) The GIC was formed in 64. (a) International Development
November 1972 consequent upon Association (IDA) , is a part of the
the nationalisation of general World Bank that helps the world’s
insurance business. The 107 poorest countries. It complements
private companies operating in the World Bank’s other lending
the field were grouped together arm— the International Bank for
Reconstruction and Development
into four - National Insurance
(IBRD) which serves middle-
Company, United India Insurance
income countries with capital
Company, Oriental Insurance
investment and advisory services.
Company and New India IDA was created in 1960.
Assurance Company, with GIC as
128 Indian Financial System

65. (c) 66. (c) Stock Exchange (1602) is


67. (a) In economics, the Laffer curve considered oldest in the world
is a hypothetical representation and was established by the
of the relationship between Dutch East India company.
government revenue raised 71. (a) Brent Crude is a major trading
by taxation and all possible classification of sweet light
rates of taxation. It is used to crude oil that serves as a major
illustrate the concept of taxable benchmark price for purchases
income elasticity – which of oil worldwide. Brent Crude
taxable income will change in is extracted from the North Sea
response to changes in the rate and comprises Brent Blend,
of taxation. Forties Blend, Oseberg and
68. (a) Deficit financing, practice in Ekofisk crudes .The Brent
which a government spends Crude oil marker is also known
more money than it receives as as Brent Blend, London Brent
revenue, the difference being and Brent petroleum. The
made up by borrowing or index represents the average
minting new funds. Although price of trading in the 25 day
budget deficits may occur for Brent Blend, Forties, Oseberg,
numerous reasons, the term Ekofisk (BFOE) market in the
usually refers to a conscious relevant delivery month as
attempt to stimulate the reported and confirmed by the
economy by lowering tax rates industry media.
or increasing government 72. (b) 73. (d)
expenditures. The influence 74. (c) The securities market is further
of government deficits upon a divided into the gilt-edged
national economy may be very market and the corporate
great. It is widely believed securities market.
that a budget balanced over 75. (c) Bombay Stock Exchange (BSE)
the span of a business cycle India’s oldest stock exchange
should replace the old ideal formally came into being
of an annually balanced in 1887 and was a regional
budget. Some economists have exchange till 2002 when it
abandoned the balanced budget became a national exchange.
concept entirely, considering 76. (a) Capital market deals with
it inadequate as a criterion of long-term finance (more than
public policy. 365 days) funds. It includes
69. (c) all facilities and institutional
70. (a) The ‘BSE SENSEX’ is a value- arrangements available for
weighted index composed of borrowing and lending of term
30 stocks and was started in 1 funds (including medium-term).
January, 1986. The origin of the 77. (d) According to the Economic
NYSE can be traced to 17 May, Survey, there are four
1792. When the Buttonwood Institutions, namely the
Agreement was signed by 24 Export Import Bank of India
stock brokers outside 68 Wall (EXIM Bank), National Bank
Street in New York under a for Agriculture and Rural
buttonwood tree. Amsterdam Development (NABARD),
Indian Financial System 129

the National Housing Bank in place, each with different


(NHB) and Small Industries conditions for eligibility and
Development Bank of India usage. Incentives (MEIS &
(SIDBI). SEIS) to be available for SEZs
78. (b) 79. (c) 80. (c) also e-Commerce of handicrafts,
81. (b) The energy company will own handlooms, books etc., eligible
and control the territory and the for benefits of MEIS. FTP
oil reserves it contains. benefits from both MEIS &
82. (c) Balance of payments (BoP) SEIS will be extended to units
is astatistical statement of all
located in SEZs.
transactions made between one
86. (d) 87. (a) 88. (d)
particular country and all other
89. (c) On 27 January 2005, the
countries during a specified
Government had decided
period of time. It also include
export-import of visible and to constitute a ‘National
invisible items. Investment Fund’ (NIF) into
83. (b) which realization from sale of
84. (d) The major thrust was to make minority shareholding of the
exports competitive, unshackle Government in CPSEs would
foreign trade from the clutches be channelised. This fund is
of a control regime and allow managed by professionals and a
import of most goods using part of it is used for use in social
only tariff as a restraint. sector – like education, health
85. (d) These schemes (MEIS and SEIS) care and employment.
replace multiple schemes earlier 90. (d)

Anda mungkin juga menyukai